There is a semicircular arc, of radius $R$, mass $M$, which is placed on a ground with its diameter initially vertical, and released.The friction is sufficient to prevent slipping. Find the angular velocity and angular acceleration of the arc when it has rotated by $\theta$.
Easy Math Editor
This discussion board is a place to discuss our Daily Challenges and the math and science related to those challenges. Explanations are more than just a solution — they should explain the steps and thinking strategies that you used to obtain the solution. Comments should further the discussion of math and science.
When posting on Brilliant:
*italics*
or_italics_
**bold**
or__bold__
paragraph 1
paragraph 2
[example link](https://brilliant.org)
> This is a quote
\(
...\)
or\[
...\]
to ensure proper formatting.2 \times 3
2^{34}
a_{i-1}
\frac{2}{3}
\sqrt{2}
\sum_{i=1}^3
\sin \theta
\boxed{123}
Comments
@Azimuddin Sheikh @Ram Mohith Here is my full solution
Suppose that the semicircle begins upright (θ=0), with the center of the diameter at x=0. The semicircle then proceeds to roll without slipping.
Coordinates of center of diameter (which would be the center of the circle if the circle was full):
xC=RθyC=R
Coordinates of a point on the semicircle (θ is effectively our state variable; each point has a constant ϕ):
x=Rθ+Rsin(θ+ϕ)y=R+Rcos(θ+ϕ)x˙=Rθ˙+Rθ˙cos(θ+ϕ)y˙=−Rθ˙sin(θ+ϕ)
Velocity of a point on the semicircle:
v2=x˙2+y˙2=2R2θ˙2+2R2θ˙2cos(θ+ϕ)
Infinitesimal mass:
dm=Mπdϕ=πMdϕ
Infinitesimal kinetic energy:
dE=21dmv2=21πMdϕ[2R2θ˙2+2R2θ˙2cos(θ+ϕ)]=πMR2θ˙2[1+cos(θ+ϕ)]dϕ
Total kinetic energy:
E=πMR2θ˙2∫0π[1+cos(θ+ϕ)]dϕ=πMR2θ˙2[π−2sinθ]
Infinitesimal gravitational potential energy:
dU=dmgy=πMgdϕ[R+Rcos(θ+ϕ)]=πMgR[1+cos(θ+ϕ)]dϕ
The gravitational potential energy integral is the same as the kinetic energy integral, except for the constant:
U=πMgR[π−2sinθ]
System Lagrangian:
L=E−U=πMR2θ˙2[π−2sinθ]−πMgR[π−2sinθ]
Equation of motion:
dtd∂θ˙∂L=∂θ∂L
Evaluation step one:
∂θ˙∂L=π2MR2θ˙[π−2sinθ]
Evaluation step two:
dtd∂θ˙∂L=π2MR2[θ˙(−2cosθθ˙)+θ¨(π−2sinθ)]=−π4MR2cosθθ˙2+2MR2θ¨−π4MR2sinθθ¨
Evaluation step three:
∂θ∂L=πMR2θ˙2[−2cosθ]−πMgR[−2cosθ]=−π2MR2θ˙2cosθ+2πMgRcosθ
Equating:
−π4MR2cosθθ˙2+2MR2θ¨−π4MR2sinθθ¨=−π2MR2θ˙2cosθ+2πMgRcosθ2MR2θ¨−π4MR2sinθθ¨=π2MR2θ˙2cosθ+2πMgRcosθRπθ¨−2Rsinθθ¨=Rθ˙2cosθ+gcosθ
Final result:
θ¨=Rπ−2RsinθRθ˙2cosθ+gcosθ
Here is a plot of theta vs time (generated from numerical integration) for a semicircle diameter of 2 meters (the height of a tall man). The period is about 3.5 seconds, which seems reasonable for an object of that size. You can see that it rocks back and forth between θ=0 (standing on one end of the diameter) and θ=π (standing on the other end).
Log in to reply
Outstanding sir got it now , thx very much. Pls see the new discussion note I started of collisions. Pls see
Log in to reply
If it's alright with you, I'll post a problem based on this one (the semicircle)
Log in to reply
Is this question same as that one in Mechanics
Log in to reply
Yeah was same
I got the value of angular velocity as, ω=23Rg(1−cosθ) but I don't know whether it is correct or not. Here is the way I did it.
As the friction is sufficient to prevent slipping the semicircular disk can be assumed to be performing pure rotation about the bottom most point let's say "O". Now, to find ω I used the concept of the law of conservation of energy. The energy changes that take place here are only rotational kinetic energy and change in potential energy. To find the potential energy first we have to calculate by how much height did the center of mass displaced. By fixing the initial position of center of mass as datum line after rotating through θ the center of mass will move down by h and I found it to be h=R(cosθ−1) which will be negative since the center of mass moved below the datum line. After rotating through θ the disk will acquire an angular velocity ω so it will have a rotational kinetic energy of 21Ioω2 where moment of inertia of the semicircular disc about point "O" is Io=2mR2+mR2=23mR2 by parrallel axis theorem. By law of conservation of energy, Total final energy21Ioω2+mgh21Ioω2+mgR(cosθ−1)21Ioω2=mgR(1−cosθ)43mR2ω2=mgR(1−cosθ)ω2=3R4g(1−cosθ)∴ω=23Rg(1−cosθ)=Total initial energy=0=0
I think this may be wrong so please inform me if it is.
Log in to reply
Did you account for the fact that the center of mass is not on the diameter?
Log in to reply
Ok. I forgot about that. But is the procedure correct.
Log in to reply
Log in to reply
(@Steven Chase) sir can u check this out
Log in to reply
This looks like an interesting one. I'll take a look at it
Here is the result after the first derivation. θ is the angle we care about, and ϕ is another angle that I used to do the integrals. I will derive it again tomorrow, and if I get the same thing, I will post the full solution in a separate note. The bottom line seems to be that the differential equation requires numerical integration to solve. Even a small angle approximation doesn't make things any better.
θ¨=Rπ−2RsinθRcosθθ˙2+gcosθ
Log in to reply
Also, I have another doubt. Will the answer vary whether we use a full arc or semicircular arc.
Log in to reply
Log in to reply
@Steven Chase sir , can we do it from bottommost part , taking torque of mg and equating it to I alpha .( between note that bottommost point is non inertial frame) ?? Also what will be the angular velocity if done by my method ) ??
Log in to reply
Log in to reply
@Steven Chase sir @Ram Mohith https://goiit.com/t/doubt-from-mechanics/39565/15?u=azimuddin_sheikh in this link
Also pls see this problem too :Log in to reply
Log in to reply
Log in to reply
Log in to reply
@Steven Chase sir , can we do it from bottommost part , taking torque of mg and equating it to I alpha .( between note that bottommost point is non inertial frame) ?? Also what will be the angular velocity if done by my method ) ?? Between from bottommost point what we have to do when we r now solving in non inertial frame ?? Is my approach correct?? @Ram Mohith bro leaving those two flaws r u sure its correct as such bottommost point is non inertial frame?? Check this link for solution to this problem : http://artofproblemsolving.com/community/c164h1785345p11808151 ( what he wrote in A5, is it correct,is it so easy )?? Steven chase sir and ram Mohith bro
Log in to reply
Log in to reply
Log in to reply